Today a new Explorer costs $39,900. In 1990, Explorers cost $24,000. What is the price relative?

Answers

Answer 1

The price relative of a new Explorer compared to the cost of Explorer in 1990 is 1.66.

What is the price relative?

The price relative is the ratio of the price of the Explorer today to the price of the Explorer in 1990. In order to determine the price relative, divide the price of the Explorer today to the price of the Explorer in 1990.

Division is the operation that is used to determine the quotient of two or more numbers. The sign that is used to represent division is ÷.

Price relative = price of the Explorer now / price of the Explorer in 1990

$39,900 / $24,00 = 1.66

To learn more about division, please check: https://brainly.com/question/13281206

#SPJ1


Related Questions

Solve the following inequality algebraically. [tex] |x - 1| \leqslant 13[/tex]

Answers

lx-1 l ≤ 13

There are 2 solutions:

x-1 ≤ 13

and

x-1 ≥ -13

Solve each one

x-1 ≤ 13

add 1 to both sides of the equation

x-1+1≤ 13+1

x ≤ 14

x-1 ≥ -13

Add 1 to both sides:

x-1+1 ≥-13+1

x ≥ -12

-12 ≤ x ≤ 14

— 3х + 2 = -4х + 4 need help

Answers

Given the following equation:

[tex]-3x+2=-4x+4[/tex]

You need to solve for "x" in order to find its value and solve the equation. To do this, you can follow the steps shown below:

1. You need to apply the Subtraction property of equality by subtracting 2 from both sides of the equation:

[tex]\begin{gathered} -3x+2-(2)=-4x+4-(2) \\ -3x=-4x+2 \end{gathered}[/tex]

2. Now you can apply the Additio property of equality by adding "4x" to both sides of the equation:

[tex]\begin{gathered} -3x+(4x)=-4x+2+(4x) \\ x=2 \end{gathered}[/tex]

Therefore, you get that the solution is:

[tex]x=2[/tex]

Find the average rate of change of f(x) = - 2x ^ 2 - x from x = 1 to x = 6 . Simplify your answer as much as possible .

Answers

The average rate of change of a function in the interval [a,b] is given by:

[tex]\frac{f(b)-f(a)}{b-a}[/tex]

In this case we have that a=1 and b=6; plugging these values in the formula above we have:

[tex]\begin{gathered} \frac{-2(6)^2-6-(-2(1)^2-1)}{6-1}=\frac{-2(36)-6-(-2(1)-1)}{5} \\ =\frac{-72-6-(-2-1)}{5} \\ =\frac{-78-(-3)}{5} \\ =\frac{-78+3}{5} \\ =\frac{-75}{5} \\ =-15 \end{gathered}[/tex]

Therefore, the average rate of change in the interval is -15

20 pts, precalc, see attach
If f (x) = 3x2 + 5x − 4, then the quantity f of the quantity x plus h end quantity minus f of x end quantity all over h is equal to which of the following?

Answers

The numeric value for the given expression is as follows:

[tex]\frac{j(x + h) - j(x)}{h} = \frac{4^{x - 2}(4^h - 1)}{h}[/tex]

How to find the numeric value of a function or of an expression?

To find the numeric value of a function or of an expression, we replace each instance of the variable in the function by the desired value.

In the context of this problem, the function j(x) is given as follows:

[tex]j(x) = 4^{x - 2}[/tex]

At x = x + h, the numeric value of the function is found replacing the lone instance of x by x + h as follows:

[tex]j(x + h) = 4^{x + h - 2}[/tex]

For the fraction, the subtraction at the numerator is given as follows, applying properties of exponents:

[tex]j(x + h) - j(x) = 4^{x + h - 2} - 4^{x - 2} = 4^{x - 2}(4^h - 1)[/tex]

As the x - 2 term is common to both exponents.

Just dividing by h, the numeric value of the entire expression is given as follows:

[tex]\frac{j(x + h) - j(x)}{h} = \frac{4^{x - 2}(4^h - 1)}{h}[/tex]

Which means that the third option is correct.

More can be learned about the numeric values of a function at brainly.com/question/28367050

#SPJ1

Of the 100 million acres in California,
the federal government owns 45 million
acres. What percent is this?

Answers

Step-by-step explanation:

you do know what a percent is ?

1% is the 1/100 part of a whole.

when we have

100,000,000 (one hundred million),

how many 1/100 parts of that are

45,000,000 (45 million)?

well, 45.

45,000,000 is 45/100 of 100,000,000

in other words 45%.

2) The Allen's rectangular backyard has a
perimeter of 144 feet. If the backyard is 40
feet wide, what is the area of their yard?

Answers

Answer:

1280 ft²

Step-by-step explanation:

Perimeter is the addition of all sides of the shape added together. Therefore, if the backyard is the rectangular shape you know that two of the sides are 40 feet wide. By adding both sides you get a total of 80 feet. Subtract the total from the perimeter in order to get the total of the unknown sides. 144 minus 80 is equivalent to 64. Now that you have the total of the unknown sides, divide by two in order to get the single unknown length. So, 64 divided by 2 is equal to 32. The area is the multiplication of two sides with quadrilaterals. Therefore, 40 times 32 equals an area of 1280 feet squared.

Explain why the volume of these two stacks of quarters are equal? Use Cavaliens principle in your explanation (If you need to)

Answers

Given the word problem, we can deduce the following information:

1. Two stacks of 18 pennies each are shown.

To determine if the volumes of the given stacks of quarters are equal, we note that Cavelieri's Principle states:

The volumes of two solids are equal if the areas of the corresponding sections drawn parallel to some given plane are equal.

Since each penny in both stacks has the same base area and has the same number of pennies. Therefore, the volumes of the given stacks are equal.

FV - OVSolve for FV in the scientific formula A =Twhere FV and OV are variables.FV =Pls help

Answers

FV=OV +TA

1) To solve for a variable means let that variable isolated on one side of the equation.

[tex]A=\frac{FV-OV}{T}[/tex]

2) Since we want to solve for FV, let's multiply it crossed like this

[tex]\begin{gathered} TA=FV-OV \\ -FV+TA-TA=-OV-TA \\ -FV=-OV-TA \\ FV=OV\text{ +TA} \end{gathered}[/tex]

3) So now we can solve for FV, all that's left are the figures for OV and TA.

twice a number increased by twenty is at least eighty-five. select all possible value of the number.
31
35
32
30
33
40
20
34​

Answers

The number when increased by twenty is at least 85, the possible values  of the numbers for this are: 35, 33, 40 and 34.

Given, according to the statement in the question, frame the equation:

2x+20 ≥ 85

⇒ 2x + 20 ≥ 85

⇒ 2x ≥ 85 - 20

⇒ 2x ≥ 65

⇒ x ≥ 65/2

⇒ x ≥ 32.5

hence the numbers greater than or equal to 32.5 are 35, 33, 40 and 34.

Hence the possible values of the number are 35, 33, 40 and 34.

Learn more about solving equations here:

brainly.com/question/13729904

#SPJ1

the variable y varies directly as x. when x =20, y= 12 what is the value of y when x = 15a:7b:9c:18d:25

Answers

To direct variations use a rule of three, as follow:

[tex]\begin{gathered} \frac{?}{12}=\frac{15}{20} \\ \\ ?=12*\frac{15}{20} \\ \\ ?=\frac{180}{20} \\ \\ ?=9 \end{gathered}[/tex]Then, when x=15, y is 9Answer: B.9


Solve 5 and 6
for 40 POINTS

Answers

Answer:

A= 184.4 yards

Step-by-step explanation:

couldnt figure out b

1,857.205 round each number to the place of the underlined digit.0 is the underlined digit

Answers

Answer:

1,857.21

Explanation:

In the number: 1,857.205

The digit after 0 is 5.

Since it is a number between 5 and 9, we round up to obtain:

[tex]1,857.205\approx1,857.21\text{ (to the nearest hundredth)}[/tex]

Find the minimum value of
C = 6x + 3y
Subject to the following constraints:
x > 1
y ≥ 1
4x + 2y < 32
2x + 8y < 56

Answers

Answer:

  9

Step-by-step explanation:

You want the minimum value of objective function C=6x+3y, given the constraints x>1, y≥1, 4x+2y<32, and 2x+8y<56.

Minimum

The objective function has positive coefficients for both x and y, so it will be minimized when x and y are at their minimum values. The constraints tell you these minimum values are x=1 and y=1, so the minimum value of C is ...

  C = 6(1) +3(1) = 9

The minimum value of C is 9.

__

Additional comment

The value of x cannot actually be 1, so the value of C cannot actually be 9. However x may be arbitrarily close to 1, so C may be arbitrarily close to 9.

  C = 6x +3y   ⇒   x = (C -3y)/6

The x-constraint requires ...

  x > 1

  (C -3y)/6 > 1

  C -3y > 6 . . . . . . multiply by 6

  C > 6 +3y . . . . . . add 3y

The minimum value of y is exactly 1, so we have ...

  C > 6 +3(1)

  C > 9

You will purchase snacks for the painting class. You have abudget of $40. You want to buy fruit and granola bars. Fruit costs $4 perpound, and the granola bars are $1 each. You need at least 20 granolabars. What combinations of fruit and granola bars can you buy?My variables- f-fruits g-granola

Answers

Writing inequalitiesWhat we know?

Budget: $40

Fruit: $4/pound

Granola bars: $1 each

At least 20 granolas

Let's call

f: number of pounds of fruit

g: number of granola bars

Therefore

The total cost of fruit is given by: $4 · f

The total cost of granola is given by: $1 · g

The total cost of everything is given by: $4 · f + $1 · g

Since budget: $40 then

$4 · f + $1 · g = $40

4f + g = 40

g > 20 means we have at least 20 granolas

4f + g > 20 means that we have at least 20 granolas, that cost $20, since 4f + g = 40 and 40 > 20

On average, Peter goes through three fish hooks in order to catch 7 fish. How many hooks can he expect to use if he needs to catch 189 fish?

Answers

By solving a proportional relation, we conclude that he needs 81 hooks to catch 189 fish.

How many hooks can he expect to use if he needs to catch 189 fish?

We assume there is a proportional relationship of the form:

F = k*H

where:

F = number of fish.k = constant of proportionality.H = number of hooks.

We know that with 3 hooks he catches 7 fish, then we can replace that:

7= k*3

7/3 = k

So the proportional relation is:

y = (7/3)*x

Then if he wants to get 189 fish we can write:

189 = (7/3)*x

And solve this for x:

189*(3/7) = x =81

He will need 81 hooks.

Learn more about proportional relationships:

https://brainly.com/question/12242745

#SPJ1

please slove this for me-3(x+1)<15

Answers

-3(x+1)<15 ​

Divide both-side of the inequality by -3

(x + 1) > -5

subtract 1 from both-side of the inequality

x > -5 - 1

x > -6

Answer:

Step-by-step explanation:

-3(x+1)<15

Divide the inequality by -3, so we need to change the sides of the less than into greater than

x+1 > -5

x > -5 - 1

x > -6

Dog Owners5. A city council wants to know if residents would like a dogpark. They sent a survey to every household in the city.The results of those who responded are shown in the tableat the rightNumbexof Dogs inHouseholdNumber ofHouseholds0513a What is an appropriate first step in finding the experimentalprobability that a household has 2 or more dogs?12182 or more129Find the product of the number of households withone dog and the number with two or more dogs.Find the difference of the number of households with twoor more dogs and the number with no dogs.© Find the sum of the number of households for each category.Find the difference of the number of households with no dogsand the number with one dog or more.b. What is the experimental probability that a householdhas 2 or more dogs?

Answers

Given:

A table represents a survey to know if residents would like a dog.

a) What is an appropriate first step in finding the experimental probability that a household has 2 or more dogs?

The first step is to find the total households

So, the answer will be option C

Find the sum of the number of households for each category.

b) What is the experimental probability that a household has 2 or more dogs?

First, the total number of households = 513 + 218 + 129 = 860

And the number of households has 2 or more dogs = 129

So, the probability = 129/860 = 0.15 = 15%

So, the answer will be 15%

Help me pretty please I need help

Answers

Answer:i have no idea good luck

Step-by-step explanation:

Answer:

4,3 DONT BE MAD IF IM WRONG

six men can complete a certain work in 20 days .how many men are required to complete the same work in 12 days ?

Answers

6 men complete the work in 20 days.

In 1 day it takes:

6 x20 = 120 men

You need to do the same work done in 20 days, in one day. (more manpower)

So, to finish the work in 12 days:

120 men / 12 days = 10 men

1118 11 + 600-1110
[tex]12 \times (5 + 4)[/tex]

Answers

Answer:

111811 + 600 - 1110

111871 - 1110

= 110761

12 × (5+4)

60+4

= 64

Which of the following is equivalent to 1,000,000?

Answers

The statement that  is equivalent to 1, 0 0 0, 0 0 0 is: C. 10 power of 6.

Determining the equivalent number

Given digit:

1,0 0 0,0 0 0

Hence,

When something  is equivalent to another things it implies or means  that both of them are the same  or correspond.

We know that 1,000,000 can be re- write as 1 Million and  1000 Thousands.

Now let determine or find the equivalent of 1,000,000 using scientific notation

Scientific notation  = 1,000,000

Scientific notation = 10^6

Based on the above  10^6 is equivalent to 1,000,000. We can justified it by pressing 10^6 on our calculator which will in turn gives us 1,000,000.

So,

1,000,000 = 10^6

Therefore we can conclude that the correct option is C based on the fact that 10  raise to power of 6 is the same as 1,0 0 0,0 0 0.

Learn more about equivalent expression here: https://brainly.com/question/10764816

#SPJ1

The complete question is:

Which expression is equivalent to 1,000,000?

A .5 power of 10. B .6 power of 10. C. 10 power of 6. D. 10 power of 5

There are two box containing only yellow and black pens

Answers

SOLUTION; Concept

Step1: Identify the giving information in the question

BOX A contains

[tex]\begin{gathered} 9\text{ yellow pens} \\ 6\text{ Black pens } \end{gathered}[/tex]

BOX B contains

[tex]\begin{gathered} 9\text{ yellow pens } \\ 11\text{ black pens} \end{gathered}[/tex]

Step2: Find the probability of each event

Event 1: Choosing a green pen from the Box B

[tex]\begin{gathered} \text{ Since there is no gr}een\text{ pen in the box, then probability of choosing a gr}en\text{ box in Box B is 0} \\ \text{then probability of choosing a gr}en\text{ box in Box B is 0} \\ Pr(E1)=0 \end{gathered}[/tex]

Event 2: Choosing a black pen from the Box B

[tex]P(E2)=\frac{11}{9+11}=\frac{11}{20}=0.55[/tex]

Event 3: Choosing a yellow or black pen from the Box A

Since Box A contains only a yellow or black pen then the probability is

[tex]Pr(E3)=1[/tex]

Event 4: Choosing a yellow pen from box A

Since there are 9 yellow pens in box A, the probability of choosing the yellow pen is

[tex]Pr(E4)=\frac{9}{9+6}=\frac{9}{15}=0.6[/tex]

Probability describes the likelihood of the event.

Hence From least likely to most likely the occurrence of the event is arranged as follows according to the probability of each event

[tex]\text{Event }1\rightarrow\text{ Event 2}\rightarrow\text{ Event 4}\rightarrow\text{ Event 3}[/tex]

Mary’s credit card company charges 16% interest on her outstanding credit card balance each month. Her minimum payment is $20 each month. Mary’s credit card bill is $70 in January. Mary only pays the minimum amount each month, and she does not spend any additional money on her credit card. How long, in months, will it take her to pay off her bill from January?

Answers

Answer:

Step-by-step explanation:

ionoknionoinponpoihj

Answer:

Step-by-step explanation:

tipptrptprprprptprtprptpprtprtpprtprptprtprptprtprptr

A textbook store sold a combined total of 217 sociology and history textbooks in a week. The number of history textbooks sold was 77 less than the number of
sociology textbooks sold. How many textbooks of each type were sold?

Answers

Answer:

there were 246 chemistry textbooks and 169 psychology textbooks sold

Step-by-step explanation:

Let

c = the number of chemistry textbooks sold

p = the number of psychology textbooks sold

a textbook store sold a combined total 415 chemistry and psychology textbooks in a week

c + p = 415

the number of chemistry textbooks sold was 77 more than the number of psychology textbooks sold

c = 77 + p

by solving the system of equations

c + p = 415

c = 77 + p

we find

c = 246 chemistry textbooks

p = 169 psychology textbooks

which function is best represented by this graph?A) f(x) = -3x² + 2x + 1 B) f(x) = 3x² + 2x - 4C) f(x) = x² + x - 4D) f(x) = x² + 2x - 3

Answers

Answer:

From the graph, the roots of the equation are given below as

[tex]\begin{gathered} x=-3 \\ x=1 \end{gathered}[/tex]

These are the points where the curve cuts the x-axis

The y-intercepts of the graph is given below as

[tex](0,-3)[/tex]

Concept:

To find the equation of the graph, we will use the formula below

[tex]\begin{gathered} f(x)=a(x+3)(x-1) \\ x=0,y=-3 \\ -3=a(0+3)(0-1) \\ \frac{-3}{-3}=\frac{-3a}{-3} \\ a=1 \end{gathered}[/tex]

Hence,

By substituting the values, we will have

[tex]\begin{gathered} f(x)=a(x+3)(x-1) \\ f(x)=1(x+3)(x-1) \\ f(x)=x^2-x+3x-3 \\ f(x)=x^2+2x-3 \end{gathered}[/tex]

Hence,

The function of the graph is

[tex]f(x)=x^2+2x-3[/tex]

OPTION D is the right answer

Last year Collin's salary was $45,000. Because of furlough days, this year his salary was $30,000. Find the percent decrease.Round to the the nearest tenth of a percent

Answers

ANSWER:

33.3%

STEP-BY-STEP EXPLANATION:

We can calculate the value of the percentage starting from the following equation:

[tex]45000-x\cdot45000=30000[/tex]

We solve for x:

[tex]\begin{gathered} -x\cdot45000=30000-45000 \\ x=\frac{15000}{45000} \\ x=\frac{1}{3}=0.333 \\ \text{ in the percent form:} \\ 0.333=33.3\text{\%} \end{gathered}[/tex]

The percentage that decreased was 33.3%

What is the P(A and B) if P(A) = 1/2 and P(B) = 2/7, where A and B are independent events?5/81/71/121/2

Answers

EXPLANATION:

To calculate the number of independent events that occur, the product of the probabilities of the individual events occurring must be calculated.

Therefore if A and B are independent events then:

P (A and B) = P (A) • P (B)

The exercise is as follows:

[tex]\begin{gathered} \frac{1}{2}\times\frac{2}{7}=\frac{2}{14};\text{ Now }we\text{ must take }square\text{ r}oot; \\ \frac{2}{14}=\frac{1}{7} \\ \text{ANSWER: }\frac{1}{7} \end{gathered}[/tex]

NOTE:

To obtain the product of two fractions, the numerators must be multiplied with each other and the denominators must also be multiplied with each other.

Write the point slope form of the line satisfying the given conditions. Then use the point slope form of the equation to write the slope intercept form of the equation. Slope=7Passing through (-6,1)

Answers

Ok, so

The point slope form of the line is given by the following formula:

[tex](y-y_1)=m(x-x_1)[/tex]

Where

[tex](x_1,y_1)[/tex]

Is a point of the line, and m is the slope.

If we replace our values:

Slope = 7

Point = (-6, 1)

We obtain that the equation is:

[tex]\begin{gathered} (y-1)=7(x-(-6)) \\ (y-1)=7(x+6) \end{gathered}[/tex]

To find the slope intercept form of the equation, we distribute in the brackets:

[tex]\begin{gathered} y-1=7x+42 \\ y=7x+43 \end{gathered}[/tex]

And the equation of our line in the slope intercept form will be:

y=7x+43

HELP PLEASE ASAP!!!!!!!!!!

Answers

Answer:

a b

Step-by-step explanation:websites and etc helped.

A school offers band and chorus classes. The table shows the percents of the 1200 students in the school who are enrolled in band, chorus, or neither class. How many students are enrolled in both classes?

Class Enrollment
Band 34%
Chorus 28%
Neither 42%

Answers

168 students are enrolled in both classes.

This is a problem from set theory. We can solve this problem by following a few steps easily.

First of all, we have to calculate the students present in both classes.

Student present in both classes = the total student - the student not enrolled in both classes.

So the percentage of the students enrolled in both classes or any of one class is ( 100% - 42% ) = 58%.

Now, the students only enrolled in chorus class is ( 58% - 34%) = 24%

So, the students who joined both classes is ( 28%- 24%)= 4%

The total student is 1200, then 4% of the total student is

( 1200 × 14 )/100 = 168 students.

To know more about set theory visit,

https://brainly.com/question/13205223?referrer=searchResults

#SPJ9

Other Questions
In triangle JKL, tan(b) = 3/4 and sin(b) = 3/5 If triangle JKL is dilated by a scale factor of 3, what is cos(b)? Rosie buys her favorite hot chocolate from a hot chocolate shop. She pays $5.00 and gets a 20 oz. cup. What is the unit rate price per ounce for her hot chocolate?(1 point) Are the triangles congruent using AAS?TrueFalse considering a model earth with no continents and with no rotation but heated like the real earth. the warm air around the equator and cold air at the poles. imagine that the university of minnesota introduces a radically new kind of grading system. now, when you take a class, every minute that you spend reading and preparing for that class can be recorded accurately. for each ten minutes you spend studying, you earn a point. you can accumulate as many points as you want just by spending more time studying for class. at the end of the semester, your grade will be based on how much time you have spent studying. this radical grading system is based on what kind of reinforcement schedule? group of answer choices Find the domain of f(x)=x-3/7x. All real numbers except -3 All real numbers except 0 All real numbers except 7 All real numbers except 3 A group of students is arranging squares into layers to create a project. The first layer has 4 squares. The second layer has 8 squares. Which formula represents an arithmetic explicit formula to determine the number of squares in each layer? hat is the effect of tariff on a particular product for the country imposing the tariff? a. increases domestic production of the product b. decreases the deadweight cost of the country c. increases domestic consumption of the product d. decreases government trade revenues Consider the function f(x) = (x+4)(x+2), Dilate f(x) by x to create a new function of a higher degree,a. Write the dilation of f(x) as g(x).b. Sketch the graph of g(x).c. identify the zeros of the graph of the function g(x) suppose that 18 inches of wire cost 72 cents. using the same rate how much in cents will 36 inches cost? The measure of three angels of a triangle are 43, 70, and x what is the value of x PLS HELPLisa needs at least 870 gigabytes of storage to take pictures and videos on her upcoming vacation. She checks and finds that she has 200 gigabytes available on her phone. She plans on buying additional memory cards to get the rest of the storage she needs. Lisa is going to use memory cards that each hold 256 gigabytes.Let C represent the number of memory cards that Lisa buys.Which inequality describes this scenario? What's the distance between -9 and 23 what variable expression should you subtract from both sides of the equation so that you are left with no variables on the right hand side Fill in the chart with the missingcause or effect.CauseA newdynasty ofpharaohs gainpower in theMiddleKingdom.Effect. determine which direction a countrys ppc will shift as a result of these major events. the country is hit by a natural disaster that destroys its infrastructure. the government starts 50 new medical clinics in rural areas. there is an acute shortage of teachers in several of the countrys states. seven computer hardware companies set up new manufacturing units in the True or false? In deductive thinking, you start with a given set of rules and cinditions and determine what must be true as a consequence An auditor would vouch inventory on the inventory status report to the vendors invoice to obtain evidence concerning managements balance assertions about. The mean of 40,50 and 90 is P. If the mean of 40,50,90 and P is Q. What is the value of Q Hello can someone please help me.